- #1
MathematicalPhysicist
Gold Member
- 4,699
- 373
Does anybody know how prove the claim in my question here:
http://theoreticalphysics.stackexchange.com/questions/643/a-question-from-ticcatis-red-qft-textbook
thanks, any hints?
http://theoreticalphysics.stackexchange.com/questions/643/a-question-from-ticcatis-red-qft-textbook
thanks, any hints?
Last edited by a moderator: